You are on page 1of 27

ENGINEERING Solved Papers

MATHEMATICS GATE 2022

Q. 4. Consider the following recursive iteration


2022 scheme for different values of variable P with
the initial guess x1 = 1:
CIVIL ENGINEERING (CE) P1
1 P
xn +1 =  xn +  , n= 1, 2, 3, 4, 5
Q. 1. Consider the following expression: 2 xn 

z sin ( y + it ) + cos ( y − it )
= For P = 2, x5 is obtained to be 1.414, rounded-
off to three decimal places. For
where z, y, and t are variables, and i= −1
P = 3, x5 is obtained to be 1.732, rounded-off
is a complex number. The partial differential
to three decimal places.
equation derived from the above expression
If P  =  10, the numerical value of x5 is
is
_________ (round off to three decimal places).
∂2 z ∂2 z 2 2
(a) 2
+ 2 =0 (b) ∂ z − ∂ z =0 Q. 5. The Fourier cosine series of a function is
∂t ∂y ∂t 2
∂y 2 given by:
∂z ∂z ∂z ∂z ∞
(c) −i =
0 (d) +i =
0 f ( x ) = ∑ fn cos nx
∂t ∂y ∂t ∂y
n = 0
Q. 2. For the equation For f(x) = cos4x, the numerical value of
3/2 (f4 + f5) is ______ (round off to three decimal
d3 y  dy 
+ x  + x2 y =
0 places).
dx 3
 dx 
Q. 6. Let max {a, b} denote the maximum of two
the correct description is real numbers a and b.
(a) an ordinary differential equation of order Which of the following statement(s) is/are
3 and degree 2. TRUE about the function
(b) an ordinary differential equation of order f (x) = max{3 − x, x − 1} ?
3 and degree 3. (a) It is continuous on its domain.
(c) an ordinary differential equation of order (b) It has a local minimum at x = 2.
2 and degree 3. (c) It has a local maximum at x = 2.
(d) an ordinary differential equation of order (d) It is differentiable on its domain.
3
3 and degree . Q. 7. Consider the differential equation
2
dy
Q. 3. The matrix M is defined as = 4 (x + 2) − y
dx

1 3
M= For the initial condition y = 3 at x = 1, the
4 2 
 value of y at x = 1.4 obtained using
and has eigenvalues 5 and − 2. The matrix Q Euler’s method with a step-size of 0.2 is
is formed as _________. (round off to one decimal place)
Q =M 3 − 4M 2 − 2M Q. 8. A set of observations of independent variable
Which of the following is/are the eigenvalue(s) (x) and the corresponding dependent
of matrix Q ? variable (y) is given below.
(a) 15 (b) 25 x 5 2 4 3
(c) –20 (d) –30
y 16 10 13 12
2OSWAAL GATE Year-wise Solved Papers engineering Mathematics
Based on the data, the coefficient a of the to two decimal places).
linear regression model Q. 15. Consider a sphere of radius 4, centered at
y = a + bx the origin, with outward unit normal n̂ on
is estimated as 6.1. its surface S. The value of the surface integral
The coefficient b is ______________. (round off
 2 xi + 3 y j + 4 zk 
to one decimal place) ∫∫s  nˆ dA is __________
4π 
Chemical Engineering (CH)
(rounded off to one decimal place).
12
Q. 9. The value of (1 + i) , where i= −1 , is dy
Q. 16. The = xy 2 + 2 y + x − 4.5 with the initial
(a) −64i (b) 64i dx
(c) 64 (d) −64 condition y (x = 0) = 1 is to be solved using a
predictor-corrector approach. Use a predictor
 x 1 3
based on the implicit Euler’s method and a
Q. 10. Given matrix A =  y 2 6  , the ordered pair corrector based on the trapezoidal rule of
 3 5 7  integration, each with a full-step size of 0.5.
(x, y) for which det(A) = 0 is Considering only positive values of y, the
(a) (1,1) (b) (1,2) value of y at x = 0.5 is ___________ (rounded
(c) (2,2) (d) (2,1) off to three decimal places).
CIVIL ENGINEERING (CE) P2
Q. 11. Let f ( x ) = e , where x is real. The value of
−|x|

df Q. 17. Let y be a non-zero vector of size 2022 × 1.


at x = −1 is Which of the following statement(s) is/are
dx
TRUE?
(a) −e (b) e
(a) yyT is a symmetric matrix.
(c) 1 / e (d) −1 / e (b) yTy is an eigenvalue of yyT.
T
Q. 12. The value of the real variable x ≥ 0, which (c) yy has a rank of 2022.
maximizes the function f(x) = xe e–x is (d) yyT is invertible.
(a) e (b) 0 Q. 18. A pair of six-faced dice is rolled thrice. The
(c) 1 / e (d) 1 probability that the sum of the outcomes in
Q. 13. The partial differential equation each roll equals 4 in exactly two of the three
attempts is ______. (round off to three decimal
∂u  1  ∂ 2 u
=  places)
∂t  π 2  ∂x 2

Q. 19. Consider the polynomial f(x) = x3 − 6x2 +
where, t ≥ 0 and x ∈ [0,1], is subjected to the
11x − 6 on the domain S given by 1 ≤ x ≤ 3.
following initial and boundary conditions:
The first and second derivatives are f ’(x) and
u(x, 0) = sin ( πx) f ”(x).
u(0, t) = 0 Consider the following statements:
u(1, t) = 0 I.  The given polynomial is zero at the
The value of t at which boundary points x = 1 and x = 3.
u(0.5, t ) 1 II. There exists one local maxima of f(x)
= is
u(0.5, 0) e within the domain S.
III. 
The second derivative f”(x) > 0
(a) 1 (b) e
throughout the domain S.
1 IV. There exists one local minima of f(x)
(c) π (d)
e within the domain S.
Q. 14. The directional derivative of f (x, y, z) = 4x2 + The correct option is:
2y2 + z2 at the point (1, 1, 1) in the direction of (a) Only statements I, II and III are correct.

the vector v= ˆi − kˆ is _________ (rounded off (b) Only statements I, II and IV are correct.
solved paper 2022 3
(c) Only statements I and IV are correct. Statement 2: v is an eigen-vector of eA.
(d) Only statements II and IV are correct. Which one of the following options is correct?
Q. 20. P and Q are two square matrices of the same (a) Statement 1 is true and statement 2 is
order. Which of the following statement(s) is/ false.
are correct? (b) Statement 1 is false and statement 2 is
(a) If P and Q are invertible, then [PQ]–1 = true.
Q–1 P–1. (c) Both the statements are correct.
(b) If P and Q are invertible, then [QP]–1 = (d) Both the statements are false.
P–1 Q–1.
x
Q. 25. Let f ( x )   e t (t  1)(t  2)dt. Then f(x)
–1 0
(c) If P and Q are invertible, then [PQ] =
–1
P Q . –1 decreases in the interval
(a) x ∈ (1,2)
(d) If P and Q are not invertible, then [PQ]–1
= Q–1 P–1. (b) x ∈ (2,3)
(c) x ∈ (0,1)
Ê x2 x3 x4 ˆ
Q. 21. Ú Á x - + - + ˜dx is equal to (d) x ∈ (0.5,1)
Ë 2 3 4 ¯
1 0 0 
(a) (1 + x ){log (1 + x ) − 1} + constant  
=
Q. 26. Consider a matrix A 0 4 −2  .
1 0 1 1 
(b) + Constant –1 2
1 + x2 The matrix A satisfies the equation 6A = A
+ cA + dI, where c and d are scalars and I is
(c) − 1 + Constant the identity matrix.
1−x
Then (c + d) is equal to
1 (a) 5 (b) 17
(d) − + Constant
1 − x2 (c) −6 (d) 11
Q. 22. The function (x, y) satisfies the Laplace Q. 27. Let, f(x, y, z) = 4x2 + 7xy + 3xz2. The direction
equation in which the function f(x, y, z) increases most
∇2 f (x, y) = 0 rapidly at point P = (1, 0, 2) is
on a circular domain of radius r = 1 with
its center at point P with coordinates x = 0, (a) 20i + 7 j (b) 20i + 7 j + 12 k
y = 0. The value of this function on the
circular boundary of this domain is equal to 3. (c) 20i + 21k (d) 20i
The numerical value of f(0, 0) is: Q. 28. Let R be a region in the first quadrant of
(a) 0 (b) 2 the xy plane enclosed by a closed curve C
(c) 3 (d) 1 considered in counter-clockwise direction.
Which of the following expressions does not
ELECTRICAL ENGINEERING (EE)
represent the area of the region R?
Q. 23. Consider a 3 × 3 matrix A whose (i, j)th
y C
element, ai,j = (i − j)3. Then the matrix A will
be R
(a) symmetric (b) skew-symmetric
(c) unitary (d) null
x
Q. 24. eA denotes the exponential of a square matrix
A. Suppose λ is an eigenvalue and
v is the corresponding eigen-vector of matrix
(a) ∫∫ dxdy
R
(b) ∫
C
xdy

A. 1
∫ 2 ∫ C
(c) ydx (d) ( xdy − ydx )
Consider the following two statements: C

Statement 1: eλ is an eigenvalue of eA.


4OSWAAL GATE Year-wise Solved Papers engineering Mathematics

Q. 29. Let E ( x , y , z ) = 2 x 2ˆi + 5 yjˆ + 3 zkˆ . The value (c) =
L32
1
,= =
U 33 2, x1 0
  2
V
( )
of ∫∫∫ ∇ ⋅ E dV , where V is the volume
1 1
(d) L = − , U 33 =
− , x1 =
0
enclosed by the unit cube defined by 0 ≤ x ≤ 1, 32
2 2
0 ≤ y ≤ 1, and 0 ≤ z ≤ 1, is
Q. 34. Which of the following is/are the
(a) 3 (b) 8
eigenvector(s) for the matrix given below?
(c) 10 (d) 5
 −9 −6 −2 −4
Computer Science (CS)  −8 −6 −3 −1
 
Q. 30. Consider the following two statements with  20 15 8 5
respect to the matrices Am×n, Bn×m, Cn×n and  32 21 7 12 

Dn×n.
Statement 1: tr(AB) = tr(BA) (a)  −1 (b)  1 
Statement 2: tr(CD) = tr(DC)  1  0
   
where tr() represents the trace of a matrix.  0  −1
Which one of the following holds?  1   0 
(a) Statement 1 is correct and Statement 2 is
wrong. (c)  −1 (d)  0 
(b) Statement 1 is wrong and Statement 2 is  0  1 
correct.    
 2  − 3
(c) Both Statement 1 and Statement 2 are  2   0 
correct.
(d) Both Statement 1 and Statement 2 are Electronics Communication (EC)
wrong.
Q. 31. The number of arrangements of six identical Q. 35. Consider the two-dimensional vector field
   
balls in three identical bins is________. F ( x , y=) xi + y j , where i and j denote the
Q. 32. The value of the following limit is unit vectors along the x-axis and the y-axis,
_____________. respectively. A contour C in the xy- plane,
as shown in the figure, is composed of two
x
lim 2 x
horizontal lines connected at the two ends
1 − e
x→0 +
by two semicircular arcs of unit radius. The
Q. 33. Consider solving the following system contour is traversed in the counter-clockwise
of simultaneous equations using LU sense. The value of the closed path integral
  
decomposition.
x1 + x 2 − 2 x 3 =4 C
( )
∫ F (x , y ) ⋅ dx i + dy j
x1 + 3 x 2 − x 3 =7 is _________.
2 x1 + x 2 − 5 x 3 =7 y
where L and U are denoted as
 L11 0 0   U11 U12 U13 
= 
L = L L 22 0  , U  0 U 22 U 23 
 21   
 L31 L32 L33   0 0 U 33 
Which one of the following is the correct
combination of values for L32 , U33 , and x1 ? x

(a) L = 1
32 2, U 33 = − , x1 = −1
2
(b) L32 = 2, U 33 = 2, x1 = −1 (a) 0 (b) 1
(c) 8 + 2p (d) −1
solved paper 2022 5
Q. 36. Consider a system of linear equations Ax = b,
where
1 − 2 3 1 
=A = , b  
 −1 2 −3  3
This system of equations admits _________.
(a) a unique solution for x
Q. 40. A simple closed path C in the complex plane
(b) infinitely many solutions for x
is shown in the figure. If
(c) no solutions for x
2z
(d) exactly two solutions for x
∫ C z 2 − 1 dz = −iπA,

Q. 37. Consider the following partial differential
equation (PDE) where i= −1 , then the value of A is ______
(rounded off to two decimal places).
∂2 f (x , y ) ∂2 f (x , y )
a = f (x, y ) ,
+b
∂x 2 ∂y 2

where a and b are distinct positive real
numbers. Select the combination(s) of values
of the real parameters ξ and η such that
f ( x , y ) = e (ξx + 1) is a solution of thegiven PDE.

1 1
(a)
= ξ = ,η
2a 2b
1
(b)=
ξ =
,η 0
a Q. 41. The function f(x) = 8 logex – x2 + 3 attains its
minimum over the interval
(c) ξ= 0, η= 0
[1, e] at x = _________.
1 1 (Here logex is the natural logarithm of x.)
(d)=
ξ =

a b (a) 2 (b) 1
Q. 38. Consider the following wave equation, (c) e (d) 1 + e
∂ f (x, t)
2
∂ f (x, t) 2 2
2
= 10000 Q. 42. Let a, b be two non-zero real numbers
∂ t ∂x 2
and v1, v2 be two non-zero real vectors of
Which of the given options is/are solution(s)
size 3 × 1. Suppose that v1 and v2 satisfy
to the given wave equation? T
v1T v1 1, and v2 v2 = 1 . Let A be
v1T v2 0,=
=
f (x, t) e − ( x − 100 t ) − ( x + 100 t )
2 2
(a) = +e the 3 × 3 matrix given by:
f (x, t) e
(b) = − ( x − 100 t )
+ 0.5e − ( x + 1000 t ) T
A = αv1 v1 + βv2 v2 + T

(c) f ( x , t )= e −( x −100 t) + sin ( x + 100t ) The eigenvalues of A are __________.
(d)
= f ( x , t ) e j 100 π( −100 x + t) + e j 100 π(100 x + t) (a) 0, a, b (b) 0, a + b, a − b

Q. 39. The bar graph shows the frequency of the α+β


(c) 0, , αβ (d) 0, 0, α 2 + β 2
number of wickets taken in a match by a 2
bowler in her career. For example, in 17 of her Q. 43. Consider the following series:
matches, the bowler has taken 5 wickets each. ∞
nd
The median number of wickets taken by the ∑c n
bowler in a match is __________ (rounded off to n=1

one decimal place). For which of the following combinations of c,


d values does this series converge?
(a) c = 1, d = −1 (b) c = 2, d = 1
(c) c = 0.5, d = −10 (d) c = 1, d = −2
6OSWAAL GATE Year-wise Solved Papers engineering Mathematics
Q. 44. The value of the integral (c) T(x, y) = −x + y

∫∫ 3 ( x )
2 (d) T(x, y) = x + xy + y
+ y 2 dxdy ,
1 2

( )
D
Q. 49. Given a function=ϕ x + y 2 + z 2 in three-
where D is the shaded triangular region 2
shown in the diagram, is _____ (rounded off to dimensional cartesian space, the value of the
the nearest integer). surface integral

y ∫∫ S nˆ ⋅ ∇ϕ dS,
where S is the surface of a sphere of unit
radius and n̂ is the outward unit normal
vector on S, is
(a) 4p (b) 3p
x 4π
(c) (d) 0
3
Q. 50. The value of the integral

 6z 
∫  2 z 4 3 2  dz
− 3z + 7 z − 3z + 5 

Mechanical ENGINEERING (ME) P1 evaluated over a counter-clockwise circular
contour in the complex plane enclosing only
Q. 45. The Fourier series expansion of x3 in the
the pole z = i, where i is the imaginary unit, is
interval −1 ≤ x < 1 with periodic continuation
(a) (−1 + i) p (b) (1 + i) p
has
(a) only sine terms (c) 2(1 – i) p (d) (2 + i) p
(b) only cosine terms Q. 51. The system of linear equations in real (x, y)
(c) both sine and cosine terms given by
(d) only sine terms and a non-zero constant  2 5 − 2α 
(x y) 
α 1 
= (0 0)
 10 2 k + 5 
Q. 46. If A =   is a symmetric matrix,
3k − 3 k + 5  involves a real parameter a and has infinitely
the value of k is ___________. many non-trivial solutions for special value(s)
(a) 8 (b) 5 of a. Which one or more among the following
options is/are non-trivial solution(s) of (x, y)
(c) −0.4 (d) 1 + 1561 for such special value(s) of a?
12 (a) x = 2, y = −2
Q. 47. The limit (b) x = −1, y = 4
 x 2 + ax + 2 π 2  (c) x = 1, y = 1
p = lim 
x →π x − π + 2 sin x  (d) x = 4, y = −2
 
Q. 52. Let a random variable X follow Poisson
has a finite value for a real a. The value of a
distribution such that
and the corresponding limit p are
Prob(X = 1) = Prob(X = 2).
(a) a = −3p, and p = p
The value of Prob(X = 3) is __________ (round
(b) a = −2p, and p = 2p
off to 2 decimal places).
(c) a = p, and p = p
Q. 53. Consider two vectors:
(d) a = 2p, and p = 3p 

a = 5i + 7 j + 2 k  
2
Q. 48. Solution of ∇ T = 0 in a square domain (0 < x

< 1 and 0 < y < 1) with boundary conditions:  
a = 3i − j + 6 k 

T(x, 0) = x; T(0, y) = y; T(x, 1) = 1 + x; T(1, y) 
Magnitude of the component of a orthogonal
= 1 + y is  
to b in the plane containing the vectors a
(a) T(x, y) = x − xy + y 
and b is __________ (round off to 2 decimal
(b) T(x, y) = x + y
places).
solved paper 2022 7

Q. 59. A polynomial ϕ ( s)= an s + an −1 s +  + a1 s + a0


n n −1
Mechanical ENGINEERING (ME) P2

of degree n > 3 with constant real coefficients
Q. 54. Given ∫ −∞
e − x dx= π. an , an −1 , … a0 has triple roots at s = −σ . Which
If a and b are positive integers, the value of one of the following conditions must be
∞ − a( x + b )
2 satisfied?
∫−∞
e dx is ________.
(a) ϕ ( s) = 0 at all the three values of s
π satisfying 3
+ σ3 =
(a) πa (b)
a
(b) ϕ ( s) = 0, dϕ ( s) = 0, and d ϕ ( s) = 0 at s = −σ
2

(c) b πa (d) b π ds 2
ds
a
(c) ϕ ( s) = 0, d ϕ ( s) = 0, and d ϕ ( s) = 0 at s = −σ
2 4

Q. 55. If f (x) = 2 ln ( e x ) , what is the area bounded 2 4


ds ds
by f (x) for the interval [0, 2] on the x-axis?
(d) ϕ ( s) = 0, and d ϕ ( s) = 0 at s = −σ
2

(a) 1 (b) 1 ds 2
2
Q. 60. For the exact differential equation,
(c) 2 (d) 4
Q. 56. F(t) is a periodic square wave function as du − xu2
= ,
shown. It takes only two values, 4 and 0, and dx 2 + x 2 u

stays at each of these values for 1 second which one of the following is the solution?
before changing. What is the constant term (a) u2 + 2x2 = constant
2
in the Fourier series expansion of F(t)? (b) xu + u = constant
( ) 1 2 2
(c) x u + 2u = constant
4
2
1 2
(d) ux + 2 x = constant
2
−3 −2 −1 0 1 2 3 4
( seconds) Q. 61. A manufacturing unit produces two products
P1 and P2. For each piece of P1 and P2, the
(a) 1 (b) 2
table below provides quantities of materials
(c) 3 (d) 4
M1, M2, and M3 required, and also the profit
Q. 57. Consider a cube of unit edge length and sides earned. The maximum quantity available
parallel to co-ordinate axes, with its centroid per day for M1, M2 and M3 is also provided.
at the point (1, 2, 3). The surface integral The maximum possible profit per day is
  
of a vector field F = 3xiˆ + 5 yjˆ + 6 zkˆ `__________.
∫ F ⋅
A
d A
over the entire surface A of the cube is ______. M1 M2 M3 Profit per
piece (`)
(a) 14 (b) 27
(c) 28 (d) 31 P1 2 2 0 150
P2 3 1 2 100
Q. 58. Consider the definite integral
2
Maximum quantity 70 50 40

∫ (4x )
+ 2 x + 6 dx. 2 available per day
1
(a) 5000 (b) 4000
Let Ie be the exact value of the integral. If the (c) 3000 (d) 6000
same integral is estimated using Simpson’s
Q. 62. A is a 3×5 real matrix of rank 2. For the set of
rule with 10 equal subintervals, the value is
homogeneous equations AX = 0, where 0 is
Is. The percentage error is defined as e = 100
a zero vector and X is a vector of unknown
× (Ie − Is)/Ie. The value of e is
variables, which of the following is/are true?
(a) 2.5 (b) 3.5 (a) The given set of equations will have a
(c) 1.2 (d) 0 unique solution.
8OSWAAL GATE Year-wise Solved Papers engineering Mathematics
(b) The given set of equations will be satisfied respectively, then |p| is ______ (in integer).
by a zero vector of appropriate size. Q. 64. Given z = x + iy, i = −1 . C is a circle of
(c) The given set of equations will have radius 2 with the centre at the origin.
infinitely many solutions.
If the contour C is traversed anticlockwise,
(d) The given set of equations will have many then the value of the integral
but a finite number of solutions
1 1
Q. 63. If the sum and product of eigenvalues of ∫
2 π ( z − i )( z + 4i )
C
dz is _______ (round off to
3 p
a 2 × 2 real matrix   are 4 and −1 one decimal place).
p q 

Answer Key
Q. No. Answer Topic Name Chapter Name
1 (a) Formulation of PDE Partial Differential Equation
2 (a) Order and Degree Differential Equation
3 (a, c) Eigen Value Matrix
4 [3.162] Iteration Method Numerical Analysis
5 [0.125] Coefficient Value Fourier Series
6 (a, b) Local Minima Maxima and Minima
7 [6.4] Euler's Method Numerical Analysis
8 [1.9] Straight Line Curve Fitting
9 (d) Complex Number Complex Number
10 (b) Matrix Matrix
11 (a) Differentiation Calculus
12 (a) Maxima and Minima Calculus
13 [1] Laplace Transform Integral Transform
14 [4.24] Directional Derivative Vector Differentiation
15 [192] Gauss Divergence Theorem Vector Integration
16 [0.875] Euler's Method, Trapezoidal Rule Numerical Analysis
17 (a, b) Eigenvalues Linear Algebra
18 [0.019] Probability Probability and Statistics
19 (b) Local Maxima and Minima Calculus
20 (a, b) Matrix Algebra Linear Algebra
21 (a) Indefinite Integrals Calculus
22 (c) Laplace Equation Partial Differential Equation
23 (b) Matrix Algebra Linear Algebra
24 (c) Eigenvalues and Eigenvectors Linear Algebra
25 (a) Increasing Decreasing Function Calculus
26 (a) Eigenvalues And Eigenvectors Linear Algebra
27 (b) Vector Calculus Calculus
28 (c) Vector Calculus Calculus
29 (c) Vector Calculus Calculus
solved paper 2022 9

30 (c) Matrix Algebra Linear Algebra


31 [7] Probability Probability And Statistics
32 [–0.5] Limits Calculus
33 (d) LU Decomposition Linear Algebra
34 (a, c, d) Eigenvalues And Eigenvectors Linear Algebra
35 (a) Vector Calculus Calculus
36 (c) Solution of Linear Equations Linear Algebra
37 (a, b) Partial Differential Equations Partial Differential Equations
38 (a, c) Wave Equations Partial Differential Equations
39 [4] Median Probability And Statistics
40 [0.5] Cauchy’s Integral Formula Complex Analysis
41 (b) Absolute Maximum And Minimum Calculus
42 (a) Eigenvalues And Eigenvectors Linear Algebra
43 (b, d) Convergence Tests Complex Analysis
44 [512] Multiple Integrals Calculus
Fourier Series for Even and Odd
45 (a) Fourier Series
Function
46 (a) Symmetric Matrix Matrix
47 (a) Limits Limits and Continuity
48 (b) Boundary Value Problems Partial Differential Equations
49 (a) Gauss Divergence Theorem Application of Vectors Calculus
50 (a) Residues Theorem Complex Number
51 (a, b) Solution of Linear Equations Matrix
52 [0.18] Poisson Distribution Probability Distributions
53 [8.32] Orthogonal Vectors Vectors
54 (b) Beta and Gamma Functions Definite Integral
55 (c) Area Bounded Definite Integral
56 (b) Value of Constant Fourier Series
57 (a) Application of Vectors Vector Calculus
58 (b) Error and Approximation Numerical Analysis
59 (b) Maxima and Minima Differential Calculus
60 (c) Exact Differential Equation Differential Equation
61 (b) Graphical Method Linear Programming
62 (b, c) Homogeneous Equations Matrix
63 [2] Eigenvalues and Eigenvectors Matrix
64 [0.2] Residues Theorem Complex Number
ENGINEERING Solved Papers
MATHEMATICS
GATE
2022

ANSWERS WITH EXPLANATIONS


1. Option (a) is correct. 3. Options (a, c) are correct.
z = sin (y + it) + cos (y – it)  …(i)
Given, M =  1 3 
z = f (y, t) then Differentiating partially with 4 2
 
respect to t both sides,
2
Then, M = M × M
∂z
= i cos( y + it ) + i sin( y − it )
∂t 1 3 1 3
=   
Again differentiate 4 2 4 2
2
∂ z 13 9 
2
=−i 2 sin( y + it ) − i 2 cos( y − it ) = 
∂t 12 16 

= sin (y + it) +cos (y – it)  …(ii) \ M3 = M 2 × M
Partial differentiate with respect to y both sides,
∂z 13 9   1 3   49 57 
= cos( y + it ) − sin( y − it ) = =     
∂y 12 16   4 2  76 68 

Again differentiate partially Q =M3 − 4M 2 − 2M
∂2 z  49 57   52 36   2 6 
=− sin( y + it ) − cos( y − it ) =  − − 
∂y 2 76 68   48 64   8 4 

= −[sin( y + it ) + cos( y − it )]  …(iii)
From (ii) and (iii), we have  49 57   54 42   −5 15 
=  − = 
∂2 z ∂2 z 76 68   56 68   20 0 
= −
∂y 2 ∂t 2 −5 − λ 15

Eigenvalue = A − λI = 0 ⇒ =
2
∂ z 2
∂ z 20 −λ
\ 2
+ =
0
∂y ∂t 2 ⇒ ( −5 − λ )( −λ ) − 15 × 20 = 0
2. Option (a) is correct.
⇒ 5λ + λ 2 − 300 = 0
It is an ordinary differential equation.
2
3 ⇒ λ + 5λ − 300 = 0
3
d y  dy  2
+ x2 y =
−x   ⇒ λ 2 + 20 λ − 15λ − 300 = 0
dx 3  dx 

As squaring both sides, we get ⇒ λ( λ + 20) − 15( λ + 20) = 0

 d3 y 
2 3 ⇒ ( λ − 15)( λ + 20) = 0 ⇒ =
λ 15, − 20
2 2  dy 
 + x y  =
x  dx 
 dx 3  Another method
2 3
Given λ 1 =5 λ 2 =−2
 d3 y  4 2
3
2 d y  dy 
 3  + x y + 2.x y 3 =x2  
 dx  Q =M3 − 4M 2 − 2M
 dx  dx

Order is 3 and degree is 2 Qλ=


1 (5)3 − 4(5)2 − 2 × 5
solved paper 2022 11
= 125 – 100 – 10 = 15 1  1 + cos 4 x 
3 2 =+ 1 + 2 cos 2 x 
Qλ 2 = ( −2) − 4( −2) − 2 × ( −2) 4  2 

= –8 – 16 + 4 = –20 1  1 + cos 4 x 1 
= + + cos 2 x 
Correct ans (a) and (c) 4  8 2 
4. Correct answer is [3.162]. 1 1 cos 4 x 1
= + + + cos 2 x
Given, x1 = 1 Initial Value 4 8 8 2
If P = 10
3 cos 4 x 1
= + + cos 2 x
1 10  8 8 2
=
x n +1  xn +   …(i)
2 xn  3 1
Put n = 1 f (x) = + 0 cos x + cos 2 x + 0 cos 3x
8 2
1 10  1  10  11 1
x 2 =  x1 +  =  1 +  = =5.5 + cos 4 x + 0 cos 5x +
2 x1  2  1 2 8

Coefficient of
n=2
1 1
1 10  1  10  f(4) +f(5) = + 0 = = 0.125
x3 =  x2 +  =  5.5 +  = 3.6590 8 8
2 x2  2 5.5 
6. Options (a, b) are correct.
n = 3 For finding intersection point
1 10  1  10  3 − x = x − 1
x4 =  x3 +  =  3.659 +  = 3.1959
2 x3  2  3.659  At x= 2

n=4 so, 3 – x and x – 1 intersects at x = 2
3 − x x < 2
1 10  1  10  
x 5 =  x4 +  =  3.196 +  = 3.162 f (x) =x − 1 x > 2
2 x4  2 3.196 
1 x=2
Hence, at P = 10 the numerical value of 
x5 = 3.162 Check for continuity
Another method (2 − ) f=
f= (2 + ) f (2)

When x → ∞ x n +1 = x n = ∞ It is continuous in its domain


1 P Check for differentiability
\ ∞
=  ∞ + 
2 ∞  −1 x < 2
f (x) = 
( ∞ 2 + P) 1 x>2
⇒ 2∞ =
∞ It is not differentiable. For maxima and minima
⇒ 2
2∞ =∞ + P 2
3–x
2
\ ∞ − P =0 ⇒ ∞ 2 =P ⇒ ∞= P
x–1
At P = 10
– +
=
∞ =
10 3.162
2 2
5. Correct answer is [0.125].
It has local minima at x = 2
2
2 2  1 + cos 2 x  7. Correct answer is [6.4].
= =
f ( x ) cos 4
x cos=x  
 2 
We have by Euler's method
1
=1 + cos2 2 x + 2 cos 2 x  yn +1 = yn + h f ( xn , yn ) ...(i)
4
12OSWAAL GATE Year-wise Solved Papers engineering Mathematics
Put n=0
( )
3
2 6 ( −1) =
3
=26 i 2 = −2 6 =− 64
y= y0 + hf ( x0 , y0 )
1

At x1 = x0 + a 10. Option (b) is correct.

= 1 + 0.2 = 1.2 x 1 3
y1 = 3 + 0.2 f (1, 3) = 3 + 0.2 [4(1+2)–3] A =0 ⇒ y 2 6 =
0

3 5 7
y1 = 3 + 0.2 [9] = 3 + 1.8 = 4.8
Put n=1 R 2 → R 2 − 2R1
y2 = y1 + hf (x1, y1)  …(ii) R → R 3 − 5R1
3
At x2 = x1 + h
x 1 3
= 1.2 + 0.2
y − 2x 0 0 =
0
= 1.4 3 − 5x 0 −8

y2 = 4.8 + 0.2 f (1.2, 4.8)
= 4.8 + 0.2[4(1.2+2) – 4.8] ⇒ x ( 0 − 0 ) − 1  −8 ( y − 2 x ) − 0  + 3 ( 0 − 0 ) =
0
= 4.8 + 0.2 [4×3.2 – 4.8] ⇒ 8 ( y − 2x ) =
0
= 4.8 + 0.2 [12.8 – 4.8] ⇒ 8y – 16x = 0
= 4.8 + 0.2 × 8 = 4.8 + 1.6 = 6.4 ⇒ y = 2x
Hence, y = 6.4 at x = 1.4 ∴ y = 2x = t
8. Correct answer is [1.9]. y = t = 2x = t
y = a + bx straight line …(i) y
 = x = t (say)
Normal equation of straight line 2
y
∑ y= an + bΣx (where n = 4)…(ii) \
2
= t ⇒ y =2t and x = t

∑ xy = aΣx + bΣx 2  …(iii) Ordered pair is (x, y) = (1, 2)


Given a = 6.1 11. Option (a) is correct.
2
x y x xy f (=
x x
x ) e= e , x is real value
5 16 25 80
 −x x < 0 
2 10 04 20 ∵ x =  
   x x ≥ 0
4 13 16 52
3 12 9 36 ∴ Differentiate with respect to x

Σy =51 Σxy =
188
df x
Σx =14 Σx 2 =
54 ⇒ =
−e
dx
51 = 4a + 14b  …(ii) dt
At x = –1 put   −1e +1 =
= −e
188 = 14a + 54b  …(iii)  dx  atx = −1
∵ a = 6.1, from (ii) 12. Option (a) is correct.
51 = 4 × 6.1 + 14b f (x) = xe e −x
⇒ 51 = 24.4 + 14b
The necessary condition for maxima and
⇒ 14b = 51 – 24.4 =26.6 maxima
26.6 df
\=b = 1.9 = 0=ie f ′( x ) 0 …(i)
14 dx
9. Option (d) is correct.
12 2 6 2
(1 + i ) = [(1 + i ) ] = [(1 + i + 2i )]
6 ⇒
df
dx
( )
= f ′( x )= x e ⋅ −e − x + e − x ⋅ ex e −1

6
=[1 − 1 + 2i ] [i 2 =−1] e e
0 ⇒ − x + ex =
∴ f ′( x ) = 0
x x
= (2
=i )6 2 6 i 6 e e x

solved paper 2022 13


−x e −ex e
= ⇒=
1
e c1 + c 2 =
0
x x x
e e x =
Put x 1=
u(1, s) 0
∴ x= e
1 sin π
13. Correct answer is [1]. 0 =c1 eπ s
+ c 2 e −π s
+
π ( s + 1)

Given partial differential equation
2 ∴ c1 eπ s
+ c 2 e −π s
0 
= …(v)
∂u = 1 ∂ u  …(i)
∂t π2 ∂x 2 From (iv) and (v)
When u = u (x, t) c=
1 c=
2 0
We have by Laplace transform 1 sin πx
L{f(t)} = F(S) u( x ⋅ s) =
π ( s + 1)
– –
L{u(x, t)} = u (x, s) = u
Taking L–1 both sides
Taking Laplace Transform both sides

 ∂u  1  ∂ u 
2
L  = 2 L 2 
L−1 u( x ⋅ s) = {
1
π
}
 1 
sin πx L−1  
 s + 1

 ∂t  π  ∂x 
1
u( x ⋅ =
t) sin πxe − t
1 ∂ u 2
π
su( x, s) − u( x, 0) =
π2 ∂x 2 Given that
1 d u 2 1 
− su =−u( x, 0) =− sin π x u  , t
2 2 2  1
π dx =
1  e
d2 u u  , 0
 
∴ − π2 su = −π sin πx …(ii) 2
dx 2
1 π
This is second order ordinary differential sin e −t
π 2 1
equations. ⇒ =
1 π e
C.S = C.F. + P.I sin e 0
π 2
2 2
(D − π s)u = −π sin πx ⇒
e −t 1
=
1 e
C.F c1 e π sx + c 2 e −π sx
=
1 1
−π sin πx 2 ⇒ =
P.I = D = − a 2 = −π 2 e t
e1
D2 − π2 s
⇒ t=1
−π sin πx 1 sin πx
= = 14. Correct answer is [4.24].
−π2 − π2 s π ( s + 1)
We have by the directional derivative of f at the
1 sin πx …(iii) point (1, 1, 1) in the direction of v = i − k is
u = C ⋅ F + P ⋅ I = c1 e π s
+ c 2 e −π s
+
π ( s + 1)
df
= grad f ⋅ v ...(i)
To evaluate c1, c2 we are given that ds
given condition u(0, t) = 0, u(1, t) = 0.  ∂f ∂f  ∂f  
grad f = ∇
= f  i + j + k
⇒ L{
=u(0, t )} 0 and =
L{u(1, t )} 0  ∂x ∂y ∂z 


= u(0, s) 0 and =
u(1, s) 0
( )
grad f = 8 xi + 4 y j + 2 zk at point (1, 1, 1)
1 sin πx
u( x, s) =c1 e π sx
+ c 2 e −π sx
+ …(iii) (grad f)(1, 1, 1) = 8i + 4 j + 2 k
π ( s + 1)
=
Put x 0=
u(0, s) 0 Direction vector v =i − 0 j − k

=
0 c1 e π s ×0
+ c 2 e −π s ×0
 …(iv)
14OSWAAL GATE Year-wise Solved Papers engineering Mathematics

v i − 0 j − k i − k
0.5
v
\ = = =  …(ii) y (0.5) – y (0) = ∫0 ( xy 2 + 2 y + x − 4.5) dx
v 1+1 2
x 0 0.5
Directional derivative of f
f(x) –2.5 2
 i − k  8 − 2
df
ds
(
= 8i + 4 j + 2 k ) ⋅
 2
=
2
=
6
2
By Trapezoidal Rule
h
y(0.5) = y(0) +  f ( x0 , y0 ) + f ( x1 , y1 )
df 6 2
\ = = 3 2 = 4.24
ds 2 0.5
= 1 + [ −2.5 + 2]
2
15. Correct answer is [192].
0.5
= 1 + × ( −0.5)
(2 xi + 3 y j + 4 zk   2
∫∫ + 3 y j + 4 zk 
(2 xinds
s 4 π ∫∫s 4π
nds
= 1 − 0.25 ==
7
0.875
We have by Gauss Divergence Theorem, 2 8
Relation between surface volume integral and
17. Option (a, b) are correct.
volume integral
    y1 
 + 3= j + 4div
zk F y 
∫∫ ∫∫ s
F ⋅xinds
(2 y∫∫∫
v
dv
nds  …(i)  2 
s 4π Let y =  y3 
 
 2 xi + 3 y j + 4 zk  
where F=  y 2022 
4π   2022 ×1

Then y = [ y1 y 2 y3 ... y 2022 ] 1×2022


 1  ∂ ∂ ∂  T
= ∫∫∫ ∇ ⋅ F dv
= ∫∫∫  (2 x ) + (3 y ) + (4 z ) dv
v 4π v  ∂z ∂y ∂z 
 y12 y1 y 2 y1 y3  y1 y 2022 
1  
4 π ∫∫∫v
= (2 + 3 + 4) dv  y 2 y1 y 22 y 2 y3  y 2 y 2022 
yyT =  y3 y1 


1 9   
= × 9 × ∫∫∫ dv = × volume of sphere.  
4π v 4π  y 2022 y1
2
 y 2022 
9 4
× π ( 4 ) = 64 × 3 = 192
3
= T
4π 3 For yy = [ zij ]2022×2022
 2 xi + 3 y j + 4 zk  Clearly zij = zji for all i,j of the matrix so

Hence, ∫∫s  4π
 n ds = 192

yyT is symmetric matrix
Option (a) – True
16. Correct answer is [0.857]. T 2 2 2 2
Now y y = [ y1 + y 2 + y3 + ... + y 2022 ]1×1  …(i)
Given differential equation T
Let λ be eigen value of yy . Then
dy (yyT) x= λ x
= xy 2 + 2 y + x − 4.5
dx T T T
Then y y( y x ) = λ( y x ) (pre multiplying by yT)
Initial condition y(0) = 1, h = 0.5
( y T y )u = λu T
for y x = u
Given x1 = x0 + h = 0 + 0.5 = 0.5
So eigenvalue of yyT is same as yTy and from
By Implicit Euler Method
(i) as yTy is (1×1) the matrix is scalar and eigen
y1 = y0 + h f(x1, y1) value for itself.
0.5 0.5
Hence yTy is one of the eigen value of yyT, and
∫0
= dy ∫ ( xy 2 + 2 y + x − 4.5) dx
0
other eigen values are zero for yyT
solved paper 2022 15
Option (b) – TRUE
12 ± 12
Also rank of yyT = rank of y = rank of yT ⇒x=
2(3)
=1 ∵ y is a non-zero column matrix
1
Option (c) – False x= 2 ± both lies within [1, 3]
T 3
As rank of yy is less than 2022, it is a singular
matrix which makes it non-invertible.  1   1 
f ′′  2 +  =6  2 +  − 12
Option (d) – False  3  3
18. Correct answer is [0.019].
 1 
Experiment: pair of six faced dice is rolled thrice f ′′  2 + >0
 3
n(S) = 6 × 6 = 36
1
Event: sum of the outcome is 4 So x= 2 + is a minima
3
E = {(1, 3), (3, 1) (2, 2)}
Statement (IV) true
n(E) = 3
 1   1 
For success f ′′  2 −  =6  2 −  − 12
 3  3
n( E)
p = P(E) =
n(S)  1 
f ′′  2 −  <0
 3
3 1
= =
36 12 1
So x= 2 − is a maxima
For failure 3
1 11 Statement (II) is true
q =1− p =1− =
12 12 Also for f  ’’(x) = 6x –12
P(exactly two of the three attempts the sum is For f  ”(x) = 0
4) ⇒ 6x –12 = 0
= 3 C2 p 2 q ⇒ x =2
2 \ f ′′( x ) < 0 for x < 2
 1  11
= 3   f  ”(x) > 0 for x > 2
 12  12
= 0.019 Statement (III) is false
19. Option (b) is correct. 20. Options (a, b) are correct.
3 2 If P and Q are two non-singular or invertible
For f ( x ) =x − 6 x + 11x − 6
matrix, then by matrix inverse property
f ′( x ) = 3x 2 − 12 x + 11
(PQ)−1 = Q −1 P −1

f ′′( x=
) 6 x − 12
−1 −1 −1
And (QP) = P Q
3 2
Here f (1) =
1 − 6(1) + 11(1) − 6 Also for singular matrices P and Q, PQ will also
f(1) = 0 be singular
33 − 6(3)2 + 11(3) − 6
f (3) = Option (d) is false
f(3) = 0 21. Option (a) is correct.
so, x = 1, x = 3 are zeros of f(x) x2 x3 x4
As log (1 + x ) =x − + − + ...
Statement I - True 2 3 4
For stationary point  x2 x3 x4 
I =∫  x − + − + ... dx
2 3 4
f ′( x ) = 0  

3x − 12 x + 11 =
0
16OSWAAL GATE Year-wise Solved Papers engineering Mathematics

= ∫ log(1 + x ) dx 0 −1 −8 
 
=A  1 0 −1
For 1 + x = u 8 1 0 
dx = du
With aij = –aji for all values of i & j in A3×3 it is a
\ I = ∫ log u du
skew-symmetric matrix.
1
= log u ⋅ ∫ du − ∫ 
u (∫ du) du by parts 24. Option (c) is correct.
For exponential of square matrix A
1
= log u ⋅ ∫ du − ∫ 
u ( ) 
∫ du  du by parts
e A =1 + A +
A2 A3
+ +
2! 3!
1
= log u ⋅ u − ∫ ⋅ u du Now with l eigenvalue v eigenvector for A
u
= u log u − ∫ du Av = λv
= u log u – u
Then A 2 v = A(A v ) = A ⋅ λv = λ ⋅ Av = λ( λv ) = λ 2 v
   = u(log u – 1)
Similarly A n v = λ n v
  I = (1 + x) {log (1 + x) – 1}
(Substituting back u=1+x) A2 A3
So, e A v =v + Av + v+ v +
22. Option (c) is correct.
2! 3!

For f (x, y) satisfying Laplace equation λ2 A3


= v + λv + v+ v +
∂2 f ∂2 f 2! 3!
2
+ =
0
∂y 2 e A =
v e λ ⋅ v ...(i)
∂x
Or in polar coordinates As we know if v is an eigenvector for an
At λt
eigenvalue & then e v = e v
1 1
frr + fr + 2 fθθ =
0 λ
r r Hence from (i) e is eigenvalue and v is
For the boundary condition with r = 3, eigenvector for eA.
0 ≤ θ ≤ 2π 25. Option (a) is correct.
f(x, y) = 3 t
Let g (t ) = e (t − 1)(t − 2),
This is possible only if f(x, y) is a constant
Clearly g(t) is continuous in open interval.
function.
So, f(x, y) = 3 By second fundamental theorem of calculus
x
And f(0, 0) = 3 For f ( x ) = ∫0 g (t ) dt
23. Option (b) is correct. We get f ′( x ) = g ( x )
 a11 a12 a13 
  ∴ f ′( x ) = e x ( x − 1)( x − 2)
Given A 3 × 3 =  a21 a22 a23  with the condition
 a31 Now for f(x) to decrease in an interval
a32 a33 
f (x) < 0

a = (i − j )3
ij
x
i.e e ( x − 1)( x − 2) < 0
 (1 − 1)3 (1 − 2)3 (1 − 3)3 
  ⇒ ( x − 1)( x − 2) < 0 ∵ e x > 0 for all x.
∴ A=
(2 − 1)
3
(2 − 2)3 (2 − 3)3 
(3 − 1)3 (3 − 2)3 (3 − 3)3  It’s clear for (x – 1) (x – 2) < 0


x ∈(1, 2)
0 ( −1)3 ( −2)3 
 3 
=
A  1 0 ( −1)3  So, f(x) decreases in interval (1, 2).
23 13 0 

solved paper 2022 17
26. Option (a) is correct. = Area of the bounded region R.

For given matrix A, the characteristic equation
is given Option (a) is true

|A – lI| = 0 As per Green’s theorem

1−λ 0 0  dQ dP 
⇒ 0 4 − λ −2 = 0
∫ P dx + Q dy= ∫∫  dx − dy  dA
C R

0 1 1−λ
For ∫ x dy with
C
= P 0,=
Q x
4 − λ −2
⇒ (1 − λ ) =0
1 1−λ  dx d0 
∫ x dy =∫∫  dx − dy  dA =∫∫ dA
⇒ (1 − λ ) {(4 − λ )(1 − λ ) − ( −2)(1)} = 0 C R R

= Area of the bounded region R.


2
{
⇒ (1 − λ ) λ − 5λ + 4 + 2 = 0 }
Option (b) is true
⇒ (1 − λ )( λ 2 − 5λ + 6) = 0
∫ y dx with P = y, Q = 0
For 
⇒ λ 2 − 5λ + 6 − λ 3 + 5λ 2 − 6 λ =0 C

⇒ λ 3 − 6 λ 2 + 11λ − 6 = 0  d0 dy 
So, by Clayley-Hamilton Theorem, we have
∫ y dx =−
∫∫  dx dy  dA =
− ∫∫ dA
C R R
3 2
A + 6A + 11A − 6 =0 = –(Area of the bounded region R)
⇒ A + 6A + 11A =
3
6 ...(i) 2
Option (c) is false
As it is given 1
6A
−1 2
= A + cA + dI
For
2∫C ( x dy − y dx ) with P = –y, Q = x
⇒ 6 =A 3 + cA 2 + dA ...(ii)
1 1  dx d( − y ) 
[On multiplying by matrix A on both sides.] 2C∫
 ( x dy − y dx=
)
2 ∫∫  dx − dy  dA
R 
Comparing (i) & (ii)
c = –6, d = 11 1
= ∫∫ (1 − ( −1)) dA=∫∫R dA
∴ c + d = –6 + 11 = 5 2 R
27. Option (b) is correct. = Area of the bounded region R.
As we know gradient of a scalar field function, Option (d) is true
points in the direction of greatest increase of 29. Option (c) is correct.
function. This direction is steepest ascent. Z
df  df  df 
∇=
f i+ j+ k G (0, 0, 1) D
dx dy dz (0, 1, 1)
F E
∇f= (8 x + 7 y + 3 z 2 )i + 7 x j + 6 xzk (1, 0, 1) (1, 1, 1)

Putting (1, 0, 2) O C (0, 1, 0)


Y
( ∇f )p = ( 8 × 1 + 7 × 0 + 3 × 2 2 ) i + 7 × 1 j + 6 × 1 × 2 k A
(0, 0, 0)

(1, 0, 0) B(1, 1, 0)
( ∇f ) p = 20i + 7 j + 12 k X

So, (20i + 7 j + 12 k ) is the direction in which For E = 2 x 2 i + 5 y j + 3 zk
f (x, y, z) increases most rapidly at P (1, 0, 2).  d 2 x 2 d 5 y d 3 z
∇=
E + +
28. Option (c) is correct. dx dy dz
As we know ∫∫ f ( x , y ) dA
R
for f ( x , y ) = 1 becomes = 4x + 5 +3

∴ ∇E
= 4x + 8
∫∫ dx dy
R
∵ dA = dx dy in cartesian plane
18OSWAAL GATE Year-wise Solved Papers engineering Mathematics

 1 1 1 33. Option (d) is correct.


∫∫∫ ∇ ⋅ E dv
=
V =
∫ ∫ ∫
z 0=y 0=x 0
(4 x + 8) dx dy dz For given set of simultaneous equation
Ax = B

(2x + 8x ) dy dz
1 1
1
= ∫ ∫
2  1 1 −2   1 0 0  u11 u12 u13 
A  1 3 =
−1  l21 0   0 u23 
0
=z 0=y 0 For= 1 u22
1 1
 2 1 −5  l31 l32 1   0 0 u33 
=
=
∫ ∫
z 0=y 0
10 dy dz
By Guess elimination method for A

∫ ([ y] ) dz
1

= 10
1 R 2 → R 2 − 1R1  l21 = 1
0
z=0
 1 1 −2 
 
0 2 1 
1

= 10 ∫ dz
 2 1 −5 
z=0 
= 10 [ z] 0
1
R 3 → R 3 − 2R1  l31 = 2

= 10  1 1 −2 
 
 0 2 1
so, ∫∫∫ ( ∇ ⋅ E ) dv = 10
v
0 −1 −1
 −1 
30. Option (c) is correct. R 3 → R 3 −   R 2  l32 = −1
 2
For any two real or complex matrices
 1 1 −2 
Pm × n, Qn × m  
Trace (PQ) = Trace (QP) 0 2 −1 
 
On this basis tr (AB) = tr (BA) 0 0 − 1 
 2
tr (CD) = tr (DC)  
So both the statements are correct.
1 1 −2 
31. Correct answer is [7].  
0 2 1
Here all six balls are identical and all three bins So U=  
are also identical 0 0 −1 
 2
So total number of arrangements is only about  
grouping these six balls in different count. This
1 0 0
could be  
(6, 0, 0), (5, 1, 0), (4, 1, 1), (4, 2, 0), (3, 3, 0), (3, 1, 2), 1 1 0
Similarly, L =  
(2, 2, 2), 2 −1
1
So possibly 7 different arrangements.  2 
 
32. Correct answer is [–0.5].
−1 −1
0 0 =
So u33 = and l32
For x → 0 + function takes i.e form 2 2
1−1 0
So applying L Hospital rule As Ax = B
1 1 ⇒ LU x =b

2 x −1 −2 x ⇒ Ly = b for Ux =y
lim+ = lim ⋅e
x →0 1 1 2 x x →0+ 2
0 − 2⋅ ⋅ e
2 x
−1
= = − 0.5
2
solved paper 2022 19

1 0 0   y   4   −7 
 1  −5 
1 1 0   y  = 7  =   is not equal to lx for any real value
   2    12 
−1
2 1   y3  7   
 2   25 
 
Option (c)
By forward substitution
 −9 −6 −2 −4   −1
1  −8 −6 −3 −1  0
y1 = 4, y2 = 3, y3 = 2 Ax =    
 20 15 8 5 2
Again    
 32 21 7 12  2
   
1 1  −3  −1
−2   x1   4  0  
    
0 2 1  x2  =  3  = =  3 0 
 6 2
−1   x3   1     
0 0    6
  2
 2 2
By back substitution \
Ax = λx with λ = 3
x = −1
3 Option (d)
x =2  −9 −6 −2 −4  0
2  −8 −6 −3 −1  1
x =0 Ax =    
1  20 15 8 5  −3 
   
−1 −1  32 21 7 12  0
=
So l32 = , u33 = , x1 0
2 2
0 0
34. Options (a, c, d) are correct. 3  
Let the given matrix be A, then we know = =  3 1 
 −9   −3 
Ax = λx where λ = eigenvalue and    
0 0
 
x = eigenvector
\
Ax = λx with λ = 3
Option (a)
 −9 −6 −2 −4   −1 35. Option (a) is correct.
 −8 −6 −3 −1  1 
) xi + y j
For the given F( x , y=
Ax =    
 20 15 8 5 0 P = x, Q = y
   
 32 21 7 12  1 By Green’s theorem
 −1  −1   dQ dP 
1   ∫ F( x , y )(dxi + dy j )= ∫∫  dx − dy  dx dy
= =  1 1  C R

0 0
     dy dx 
1
  1 = ∫∫  dx − dy  dx dy
R
\ Ax = λx with λ = 1
Option (b) = ∫∫
= 0 dxdy 0
R
 −9 −6 −2 −4   −1
 −8 −6 −3 −1  0
Ax =    
 20 15 8 5  −1
   
 32 21 7 12  0
20OSWAAL GATE Year-wise Solved Papers engineering Mathematics
Alternative: ξ2 η2
 + =
1
) xi + y j
2 2
For F( x , y=  1   1 
    
x2 y2  a   b
F( x , y=
) + could be potential fuction
2 2
Both option (a) and (b) satisfies this equation.
 df df 
So that= F i + j , which makes the field 38. Options (a, c) are correct.
dx dy
conservative f ( x , t ) e − ( x −100 t ) + e − ( x + 100 t )
Option (a) :=
2 2


∫ F(dxi + dy j ) = 0 ∂ 2 f (x , t)
= 2
2 2
e − x +200 tx −10000 t ( −2 x + 200t )2
C
∂x
−x 2
+200 tx − 10000 t 2 2
− 200 tx − 10000 t 2
36. Option (c) is correct. −2 e + e−x
For system of linear equation Ax = b 2
− 200 tx − 10000 t 2
( −2 x − 200t )2 − 2 e − x

[A|b] will be
∂ 2 f (x , t) 2 2
1 − 2 3  1 = 2
e − x + 200 xt −10000 t ( −20000t + 200 x )2
  ∂t
 −1 2 −3  3 

2 2
−20000 e − x + 200 xt −10000 t
For rank of A ∵ Row 1 = –Row 2 +e−x
2
− 200 xt − 10000 t 2
( −20000t − 200 x )2

The rows are linearly dependent.
2
− 200 xt − 10000 t 2
∴ rank A = 1  −20000 e − x
For rank of [A|b], second order minor ∂2 f ∂2 f
Substituting the values of and in given
1 1 ∂x 2 ∂t 2
= 3 × 1 − ( −1)1 = 4 ≠ 0 equation, we get
−1 3
∂ 2 f (x , t) ∂ 2 f (x , t)
= 10000
∴ rank of [A|b] = 2 ∂t
2
∂x 2
So, we have rank of A ≠ rank of [A|b] Option (a) is satisfied
Hence system of linear equation has no solution
f ( x , t ) e −( x −100 t ) + 0.5e −( x +1000 t )
Option (b): =
for x.
∂ 2 f (x , t)
37. Options (a, b) are correct. = e − x +100 t + 0.5e − x −1000 t
∂x 2
For given f ( x , y ) = e ξx +ηy
∂ 2 f (x , t)
∂f = 2
10000 e − x +100 t + 500000 e − x −1000 t
= ξ ⋅ e ( ξ+ηy ) ∂ x
∂x
∂2 f ∂2 f
∂ f 2
Substituting the values of and in
=ξ 2 ⋅ e ( ξx +ηy ) ∂x 2 ∂t 2
∂x 2 given equation, we get

∂f ∂ 2 f (x , t) ∂ 2 f (x , t)
Also, = η e ( ξx +ηy ) ≠ 10000
∂y ∂t 2 ∂x 2
∴ Option (b) is not satisfied
∂2 f
2
= η2 e ( ξx +ηy ) Option (c): f ( x , t )= e − ( x −100 t ) + sin( x + 100t )
∂y
∂2 f (x , t)
For given PDE 2
= e − x + 100 t − sin( x + 100t )
∂ t
∂2 f (x, y) ∂2 f (x, y)
a + b =
f (x, y) ∂2 f (x , t)
∂ x2 ∂y 2 = 10000 e − x + 100 t − 10000 sin( x + 100t )
2
∂t
⇒ a ξ 2 e ( ξx +ηy ) + b η2 e ( ξx +ηy ) =e ( ξx +ηy ) ∂2 f ∂2 f
Substituting the values of and in
2 2
⇒ a ξ + b η =1 ∵ e ξx +ηy ≠ 0 ∂x 2 ∂t 2
given equation, we get
This is a general equation of ellipse.
solved paper 2022 21

∂2 f (x , t) ∂2 f (x , t) 40. Correct answer is [0.5].


= 1000
∂t
2
∂x 2 2z
For g ( z ) = , we have poles at z = 1, z = –1
Option (c) is satisfied z2 − 1
Here z = 1 lies outsides the contour and z = –1
=
Option (d): f ( x , t ) e j 100 π( −100 x + t ) + e j 100 π(100 x + t )
lies inside the contour.
∂ 2 f (x , t) π 100( −100 x + t ) π 100(100 x + t )
= 1000 π2 j 200 e j + 1000 π2 j 200 e j 2z
∂x 2 So, f ( z ) = is analytical in C and z0 = –1 lies
∂ 2 f (x , t) π 100( −100 x + t ) π 100(100 x + t )
z −1
= π2 j 200 e j + π2 j 200 e j
∂t 2 within C.
∂2 f ∂2 f By Cauchy’s Integral formula
Substituting the values of 2
and 2 in
given equation, we get ∂x ∂t f ( z)
∫ z − z dz= 2 π i f ( z0 )
∂ f (x , t) 2
∂ f (x , t) 2
0

2
≠ 10000
∂t ∂x 2 2z
\ Option (d) is not satisfied.
⇒ ∫ z −1
dz =2 π i f ( −1)

z − ( −1)
39. Correct answer is [4].
Total number of matches played is given by  2 −1 
= 2 π i 
sum of the frequency  −1 − 1 
N=∑f 1 1
= 2π i ⋅ ⋅
2 −2
= 5 + 7 + 8 + 25 + 20 + 17 + 8 + 4 + 3 + 2 + 1
πi
N = 100 {even value} = −
2
No. of Frequency Cumulative
wickets Frequency 2z
0 5 5
Given
∫ z2 − 1 dz =−i π A
1 7 12 1
∴ −i π A =−i π ⋅
2
2 8 20
3 25 45 1
Hence, A= = 0.5
2
4 20 65
41. Option (b) is correct. logex
5 17 82
6 8 90 ( x ) 8 log e x − x 2 + 3
For f=
7 4 94 8
f ′( x ) = − 2 x + 0,
8 3 97 x
9 2 99 f ’(x) is not defined at x = 0, which is not in [1,e]
10 1 100 For f ’(x) = 0,
For N being even 8
− 2x =
0
1  N th
N   th x
Median
=  term +  + 1 term 
2  2 2   ⇒ 8 = 2x2
⇒ 4 = x2
1
= (50 th term + 51st term) or x = ± 2
2
x = –2 is not in [1, e), x = 2 is in [1, e]
1
= { 4 + 4} Checking f (x) at end points and critical points
2
with [1, e]
=4
22OSWAAL GATE Year-wise Solved Papers engineering Mathematics
f (1) = 8 loge 1 – 12 + 3 = 2 ∞
1
f (e) = 8 loge e – e + 3 = 3.61 2 is of the form ∑n
n=1
p
with p ≤ 1, therefore
series diverges.
f (2) = 8 loge 2 – 22 + 3 = 4.54
(b) c = 2, d = 1
Here, f (x) = 2 is minimum at x = 1.

n
42. Option (a) is correct. Series ∑2
n=1
n
a d 
b  ,   Ratio test
=Let v1 =  v2 e 
 c   f  an +1
p = lim n
n →∞ a

d 
v1 T ⋅ v2 = [ a b c ]  e  = ad + be + cf ( n + 1) 2 n
= lim n + 1 ⋅
 f  n →∞ 2 n

1 n+1
∵ v1 T v2 = 0 (given) = lim
2 n →∞ n
∴ ad + be + cf =
0 ...(i)
1
a p = with 0 ≤ p < 1, the series converges.
2
Now, v2 ⋅ v1 = [ d e f ] b  = da + eb + fc
T
  (c) c = 0.5, d = –10
 c 

n −10

2n
T
∴ v2 v1 = 0 [from (i)] Series ∑ n
= ∑n 10
n=1 0 ⋅ 5 n=1
Given, A = α v1 ⋅ v1 T + β v2 ⋅ v2 T ...(ii)
Ratio test
Then v1 = αv v1 + β v2 v2 v1 an +1
 (post multiplication by v1) p = lim n
n →∞ a

⇒ Av1 = αv1 (v1 T v1 ) + β v2 (v2 T v1 )
2n+1 n10
⇒ Av1 = αv1 {v1T v1 =1, vT2 v1 =
0} = lim ⋅
n →∞ ( n + 1)10 2n
So α is an eigenvalue with v as eigenvector.
1
10
Again A = α v v T + β v ⋅ v T  n 
1 1 2 2 = 2 lim  
n →∞
 n+1
Av2 = α v1 v1T v2 + β v2 ⋅ v2T ⋅ v2
 (Post multiplication by v2) \ p=2
T T
Av2 = α v1 (v v2 ) + β v2 (v v2 )
1 2 With p = 2, the series diverges.
(d) c = 1, d =–2
Av2 = β v2 ∞

n −2 1
So, b is an eigenvalue with v2 as eigenvector. Series ∑ n
= ∑
n=1 n
2
n=1 1
Also A will be a singular matrix so, zero could
n −2 ∞
be an eigenvalue. is of the form ∑
n=1 n
p
with p > 1, therefore
43. Options (b, d) are correct.
series converges.

nd
For series ∑c
n=1
n
44. Correct answer is [512].
For the given triangular region
(a) c = 1, d = –1
Series n −1∞ ∞
1
=
∑ 1
n 1=
n
= ∑
n 1 n
solved paper 2022 23
y 46. Option (a) is correct.
(4, 4) Symmetric Matrix:
(0, 4)
If a square matrix A is said to be Symmetric
Matrix
x
y= If A T = A A T = Transpose of A

(0, 0)
x  10 2k + 5
Given A =  
3k − 3 k + 5 
x=0 x=4
y=

 10 3k − 3 
–x

⇒ AT =  
(0, –4)
2k + 5 k + 5 
(4, –4) From equation (i)
x=4 y=x AT = A
=I 3 ∫ ∫ ( x 2 + y 2 ) dy dx  10 3k − 3   10 2k + 5
x= 0 y= −x  2 k + 5 k + 5  = 3k − 3 k + 5 
   
 
4
y 3  
x
Equating elements of both matrices, we get
∫ 
2
= 3 x y +   dx
x=0 
3 −x 
  3k – 3 = 2k + 5
3k – 2k = 5 + 3
4
 x3  x 3 
= 3 ∫  x 3 + −  −x 3 −   dx k= 8
3  3 
0  47. Option (a) is correct.
4
8x 3  x 2 + αx + 2 π 2 
= 3∫ dx Given p = lim 
3 x →π x − π + 2 sin x

0  
4 For x = π, the denominator becomes zero, thus
= 2x 4
0 for p to have a finite value. The numerator must
= 512 also be zero for x = π
At x = π π2 + aπ + 2π2 = 0
45. Option (a) is correct.
⇒ απ + 3π2 = 0
Given f(x) = x3 (odd function) – 1 ≤ x < 1
⇒ απ = −3π2 ∴ α = −3π
f(–x) = (–x)3 = –x3 = –f(x)
Now,
f ( − x ) =− f ( x )
 x 2 − 3πx + 2 π2 
p = lim  
We have by Fourier series for odd function x →π
 x − π + 2 sin x 
=a0 0=an 0=bn ? 0 
 0  Indeterminant form
 


=f (x) ∑ bn sin mπx …(i)
n =1
By L’Hospital Rule.
 2 x − 3π  2 π − 3π −π
p = lim  = = = π
 a f ( x ) dx = 2 a f ( x ) dx x →π 1 + 2 cos x 
 ∫− a ∫0   1 + 2 cos π 1 −2
 if f ( x )is even function Thus, p = p
∵ a
 f ( x ) dx = 0
 ∫− a
48. Option (b) is correct.
 if f ( x )is odd function Option (a) T( x ,1) = x − x + 1 = 1 ≠ 1 + x

Option (b) T( x , y )= x + y
Only sine terms are present. T( x ,1)= x + 1 satisfy
Option (c) T( x , y ) =− x + y
24OSWAAL GATE Year-wise Solved Papers engineering Mathematics

T( x ,1) =− x + 1 not satisfy 6i i


= =
Option (d) T( x , y ) =x + xy + y 6 + 6i i + 1
T( x ,1) = x + x + 1 = 2 x + 1 ≠ 1 + x not satisfy 6 z dz
49. Option (a) is correct.
∴ ∫ 2 z 4
− 3z + 7 z 2 − 3z + 5
3

1 2  i  2 π( −1) −2 π(i − 1)
Given,
= ϕ (x + y2 + z2 ) =
2 πi   = =
2  i + 1  (i + 1) (i + 1) × (i − 1)
∂ϕ  ∂ϕ  ∂ϕ  1 
∇=
ϕ
∂x
i+
∂y
j+
∂z
k=
2
(
2 xi + 2 y j + 2 zk ) ⇒
−2 π(i − 1) −2 π(i − 1)
i2 − 1
=
−1 − 1
 22ππ((ii−−1)
1)
∇ ϕ= xi + y j + zk= F =
=−− =π
=π((ii−−1)
1)
−−22
We have by Gauss Divergence Theorem 51. Options (a, b) are correct.
  
∫∫ n ⋅∇ϕ=
 S
dS ∫∫∫ div.F dV
= ∫∫∫ ∇.F dV
V v
For Non-Trivial solution, |A| = 0
   2 5 − 2α
⇒ ∫∫∫ ∇.∇ϕ dV
= ∫∫∫ ∇.( xi + y j + zk ) dV ⇒
α 1
= 0 ⇒ 2 − α(5 − 2α )= 0
V V

⇒ ∫∫∫ (1 + 1 + 1) dV = 3 ∫∫∫ 1 dV = 3 × Volume of ⇒ 2 − 5α + 2 α 2 = 0 ⇒ 2 α 2 − 5α + 2 = 0


V V
sphere
⇒ 2 α2 − 4 α − α + 2 = 0 ⇒ 2α ( α − 2) − 1( α − 2) = 0
4
⇒ 3 × π(1)3 = 4 π 1
3 ⇒ ( α − 2)(2α − 1) = 0 ∴ α= 2, α=
Here V is the closed region bounded by the 2
surface S. S is the surface area of sphere of unit  2 1
For α = 2, A= 
radius.  2 1
50. Option (a) is correct.  2 1
4 3 2 Thus, [ x y]   = [0, 0]
Let f ( z ) = 2 z − 3 z + 7 z − 3 z + 5  2 1
4 3 2
Here, f (i ) = 2i − 3i + 7i − 3i + 5 ⇒ 2x + 2y = 0 ⇒ x + y = 0 ⇒ x = − y
= 2(1) − 3( −i ) + 7( −1) − 3i + 5 Option (a) satisfy this condition.

2 4
= 2 + 3i − 7 − 3i + 5 For α =
1
A = 1 
2  1
=0 2 
\ z = i is a singular point
2 4
6z
Thus, ∫ 2 z 4 − 3 z 3 + 7 z 2 − 3 z + 5 dz Thus, [x y]  1  = [0, 0]
 1
 2 
 = 2 πi [|residue|z =i ] 1
2x +
y= 0 and 4x + 1y = 0
=
where Residue|z =i lim ( z − i ) f ( z ) 2
z →i
–1
⇒x= y
6z 4
= lim ( z − i )

z →i 2 z 4 − 3z3 + 7 z 2 − 3z + 5
Thus, y = –4x
6 z − 6 zi2
0  52. Correct answer is [0.18].
⇒ lim 4  form  .
z →i 2 z − 3 z + 7 z − 3 z + 5  0
3 2
 Given that P (X = 1) = P (X = 2)
By L’Hospital Rule We have by poison distribution

⇒ lim  3 12 z2 − 6i  12i − 6i e − m ⋅ mr
= 3 p( r ) = where r = 0, 1, 2, 3…
z → i 8 z − 9 z + 14 z − 3  8i − 9i 2 + 14i − 3
  r!
6i 6i P (X = 1) = P (X = 2)
⇒ =
8( −i ) − 9( −1) + 14i − 3 −8i + 9 + 14i − 3
solved paper 2022 25

e −m m e −m m2 1 π
⇒ = = π =
1! 2!
a a
2 ∴ ∞ π
⇒ m= m ∫
2

⇒ m 2 = 2m e − a( x + b ) dx =
−∞ a
1 2
2
55. Option (c) is correct.
⇒ m − 2m = 0 ⇒ m( m − 2) = 0 ⇒ m = 0, 2
1
Thus, m = 2 (Because m cannot be zero) Given, f ( x =
) 2 log( e x =
) × 2 log( e x )
2
e −2 (2)3 8 −2 =
f ( x ) x=
log e e x  ∵ log e e = 1
P(X= 3)
= = e = 0.18
3! 6
We have, area bounded by f(x) with in the
53. Correct answer is [8.32]. interval [0, 2] on x-axis.

Given a = 5i + 7 j + 2 k 2
2 2  x2 
 Therefore, ∫ f=
( x ) dx ∫0
= x dx  
 2 0
b = 3i − j + 6 k
0

1 2 1
= [2 − 0 2 ] = × 4 = 2 2
2 2
56. Option (b) is correct.
b
We have Fourier Series Expansion
q
a0 ∞
a f (t=
) + ∑ an cosnt + bn sinnt ...(i)
2 n =1
  
Magnitude of a orthogonal to b in place of a a
  Constant term in Fourier Series is 0 .
= b | a ||sin θ | 2
and
1 2 1 2

l ∫0 1 ∫0
We have =
where a0 = f (t ) dt f (t ) dt
 
a⋅b (5i + 7 j + 2 k ) ⋅ (3i − j + 6 k ) where l = 1 and period [0, 2]
=
cos θ = 
| a | ⋅| b | 5 2 + 7 2 + 2 2 ⋅ 3 2 + ( −1)2 + 6 2 1 2
1 ∫0
a0 = f (t ) dt = 1 × Area under the curve
5 × 3 + 7 × −1 + 2 × 6 20 with in [0, 2]
=cos θ =
78 ⋅ 46 78 ⋅ 46 ⇒ 1×4=4
20 a0 4
cos θ = ⇒
= θ 70.495° Constant Term = = = 2
78 ⋅ 46 2 2
 57. Option (a) is correct.
=
| a |sin θ| 78 sin(70.495°) 
Given, F = 3xi + 5 y j + 6 zk
= 8.32 We have by Gauss’s Divergence Theorem,
54. Option (b) is correct. Relation between surface integral and volume

integral

− x2
Given that e dx= π ...(i)
−∞    

∫∫S F = ⋅ d A ∫∫∫ div F ⋅ dV ...(i)

2
Find the value of e − a( x +b ) dx V
−∞
    ∂ ∂ ∂ 
div F = ∇ ⋅ F =  i + j + k  ⋅ (3xi + 5 y j + 6 zk )


a ( x + b )]2
we have e[ dx  ∂x ∂y ∂z 
−∞

let a ( x + b ) =
t =

∂x
∂ ∂
( )
(3x ) + (5 y ) + (6 z ) i ⋅ i = j ⋅ j = k ⋅ k = 1
∂y ∂z

⇒ a dx = dt 
dt div F = 3 + 5 + 6 = 14
∴ dx =
a Here V is the closed region bounded by the
∞ dt surface S.
1 ∞ −t

2
Thus, e −t
∫ e dt from equation (i)
2

−∞
a a −∞
26OSWAAL GATE Year-wise Solved Papers engineering Mathematics
   
∫ F ⋅ dA = ∫ F ⋅ n dA = ∫∫∫ 14 dV dA = n dA From (iii) and (iv), we have
∂M ∂N
= which is
A A v
∂u ∂x
⇒ 14 × Volume of cube exact differential equation.
⇒ 14 × (1)3 = 14 [volume of a cube = a3]
58. Option (b) is correct.
Solution ∫ Take u as
constant
M dx + ∫Take only x terms N du =
not contain x
C

Here the function f ( x ) = 4 x 2 + 2 x + 6 is a xx22


⇒ ⇒ uu22
 xu dx + 2du = c ⇒
2
++22uu =
=
cc
polynomial of degree 2 and the simpson's 1/3rd 22
Rule also uses a second degree polynomial for 61. Option (b) is correct.
approximation. Formulate given as an Linear Programming
The value of Ie and Is will be same. problem
i.e I e = I s Profit Maximize z = 150x1 + 100 x2...(i)
Then we have 2x1 + 3x2 ≤ 70 ...(ii)
Percentage Error e = 2x1 + x2 ≤ 50 ...(iii)
Exact Values(Ie ) − Estimated Value (I s ) 2x2 ≤ 40  ...(iv)
× 100
Exact Values(Ie ) All x1, x2 ≥ 0 (Non Negative Quantity)

I −I  By Graphical Method
Thus,
= e  e e  × 100
= 0 Convert all inequalities in equalities
 Ie 
2x1 + 3x2 = 70 ...(ii)
59. Option (b) is correct.
2x1 + x2 = 50 ...(iii)
Since ϕ(s) has triple roots at s = –s, (s + s)3 will
be one of its factors and ϕ(–s)= 0 and 2x1 = 40 ...(iv)
There will be an inflection point on the curve of x 1 , x2 = 0
ϕ(s) (Just like in the case of x3), hence, the first From equation (ii) 2x1 + 3x2 = 70
and second order derivative at s = –s will be
x1 0 35
zero.
x2 23.33 0
60. Option (c) is correct.
Given Differential Equation From equation (iii) 2x1 + x2 = 50

du − xu2 x1 0 25
=
dx 2 + x 2 u x2 50 0
It can be written as From equation (iv) 2x2 = 40
2 2
xu dx + (2 + x u) du =
0 ...(i) x2
(0, 50)
within is in the from
50
Mdx + Ndu = 0 ...(ii) 3)
2 3.3
40 (0,
where M f=
= 1 ( x , u) and N f2 ( x , u)
30
Comparing M= xu2 and N= 2 + x 2 u (5, 20) x2 = 20
(III)
We check the condition of exactness (20, 10)
B (35, 0)
∂M ∂N (0, 20) 10
=
∂u ∂x
x1
0 10 20 A 30 40 50
∂M
= 2xu ...(iii)
∂u (25, 0) (I)
(II)
∂N
and = 0 + 2xu ...(iv) Coordinate of B is the intersecting point of line
∂x
I and II
solved paper 2022 27
2x1 + 3x2 = 70 2. 
The product of the eigenvalues of the
2x1 + x2 = 50 Matrix A = The Determinant of A
– – –
2x2 = 20 3 p
⇒ −1 =
p q
⇒ x2 = 10
Therefore, 2x1 + x2 = 50 ⇒ 3q – p2 = –1
⇒ 2x1 + 10 = 50  q = 1, then 3 – p2 = –1
∴ 2x1 = 40 ∴ p2 = 3 + 1 = 4
⇒ x1 = 20 ⇒ p2 = ± 2
Coordinate of point B (20, 10) Thus, |p| = 2
Put x2 = 20 in equation 64. Correct answer is [0.2].
2x1 + 3x2 = 70, we get 1 1
2x1 + 60 = 70
Given, ∫
2 π ( z − i )( z + 4i )
c
dz ...(i)

⇒ 2x1 = 10 Poles : Taking denominator of f(z) equal to two


Thus, x1 = 5 ( z − i )( z + 4i ) =
0 ∴ z = i , − 4i
Hence, C (5, 20) and C is the circle of Radius 2. z = + i will lies
Bounded Regain OABCDO inside the circle.
Maximize z = 150x1 + 100x2 y

At A (25, 0), z = 150 × 25 + 100 × 0 = `3750 z =i


At B (20, 10), z = 150 × 20 + 100 × 10 = `4000 x
At C (5, 20), z = 150 × 5 + 100 × 20 = `2750 (2, 0)
At D (0, 10), z = 150 × 0 + 100 × 20 = `2000
80 profit maximum per day is `4000
62. Options (b, c) are correct. z = –4i
Given r(A) = 2 We have by Cauchy’s Residue Theorem,
In homogeneous equation AX = 0 ∴ ∴ BB =
=
00
∫C
f ( z )dz= 2 πi [Residues f ( z )]z = i
where A is the Coefficient matrix.
Homogeneous equations have a solution. where Residues[ f ( z=
)]z = i lim( z − i ) f ( z )
z →i

Because ρ(A) =
ρ(C) =
2 < No of unknowns 1  1  1 1
lim( z − i ) × = lim  =  =
Infinite many solutions including a zero vector z →i ( z − i )( z + 4i ) z →i  z + 4i  i + 4i 5i
of appropriate size. 1 1
2 π ∫C ( z − i )( z + 4i )
63. Correct answer is [2]. ∴ dz

We have by the properties of eigenvalues


2 πi  1  2π 1
1. The sums of the eigenvalues of Matrix = =  =
 = = 0.2.
Trace of the Matrix 2 π  5i  5 × 2 π 5
⇒ 4=3+q
∴ q=1

You might also like